subject
Physics, 04.07.2019 23:20 ellycleland16

Two thin plates (0.50 plane. one of the large plates crosses the y-axis at y = 0 cm, and the other crosses at y = -1 cm. the charge on the positive plate is +30 nc, and on the negative plate, -30 nc. a) what is the voltage from the positive plate to the negative plate? b) what is the electric potential difference from the negative plate to a point in the middle of the space between the plates? c) if the electric potential at the negative plate is 100 v, what is the electric potential at the positive plate? 'very thin') of a parallel capacitor are located parallel to the xz- m x 0.50 m x (d-f) the negative plate is now shifted 0.5 cm away from the positive plate d) what is the voltage from the positive to the negative plate? (e-f) if an electron is released from rest from very close to the negative plate, and it flies to the positive plate, e) what is the electron's change in potential energy? f)what is the electron's speed when it hits the positive plate?

ansver
Answers: 3

Other questions on the subject: Physics

image
Physics, 21.06.2019 21:20, janayshas84
An artificial satellite circles the earth in a circular orbit at a location where the acceleration due to gravity is 6.03 m/s^2. determine the orbital period of the satellite.
Answers: 3
image
Physics, 21.06.2019 21:30, jwoody5495
Although light from the sun appears white, it is actually made up of a lot of different colors. this portion of the electromagnetic spectrum, we call white light is referred to as within the electromagnetic spectrum.
Answers: 1
image
Physics, 22.06.2019 08:30, herchellann302
A40.0 l tank of ammonia has a pressure of 12.7 kpa. calculate the. volume of the ammonia if it’s pressure is changed to 8.4 kpa while its temperature remains constant.
Answers: 3
image
Physics, 22.06.2019 10:30, arigar
Aperson gives a shopping cart an initial push along a horizontal floor to get it moving, and then lets go. the cart travels forward along the floor, gradually slowing as it moves. consider the horizontal force(s) on the cart while it is moving forward and slowing. which of the following statements is correct? (a) both a forward and a backward force are acting on the cart, but the forward force is larger. (b) only a forward force is acting which diminishes with time. (c) both a forward and a backward force are acting on the cart, but the backward force is larger. (d) only a backward force is acting, no forward force.
Answers: 3
You know the right answer?
Two thin plates (0.50 plane. one of the large plates crosses the y-axis at y = 0 cm, and the other c...

Questions in other subjects:

Konu
Mathematics, 15.07.2020 21:01
Konu
English, 15.07.2020 21:01